Jump to content

Math04's Content

There have been 119 items by Math04 (Search limited from 06-06-2020)



Sort by                Order  

#737935 Đặt $b_n=\sqrt{n}(a_n-L)$ với $L=lima_n$....

Posted by Math04 on 23-03-2023 - 00:00 in Dãy số - Giới hạn

Với số thực $x$ có phần lẻ khác $\frac{1}{2}$, ta kí hiệu $<x>$ là số nguyên gần nhất với $x$. Xét dãy: $a_n=\sum_{k=1}^{n}\frac{1}{<\sqrt{k}>}-2\sqrt{n}, n=1,2...$.

a) Chứng minh dãy trên hội tụ và tìm giới hạn đó.

b) Đặt $b_n=\sqrt{n}(a_n-L)$ với $L=lima_n$. Chứng minh với mọi số thực $\alpha \in [0;\frac{1}{4}]$, luôn tồn tại một dãy con của $(b_n)$ có giới hạn bằng $\alpha$




#738040 $\frac{a^2+b^2+c^2}{ab+bc+ca}+\frac{8...

Posted by Math04 on 25-03-2023 - 15:51 in Bất đẳng thức - Cực trị

Cho $a,b,c>0$. Dùng phương pháp $p,q,r$ hãy chứng minh: $\frac{a^2+b^2+c^2}{ab+bc+ca}+\frac{8abc}{(a+b)(b+c)(c+a)}\geq2$




#738044 $\frac{a^2+b^2+c^2}{ab+bc+ca}+\frac{8...

Posted by Math04 on 25-03-2023 - 17:07 in Bất đẳng thức - Cực trị

mình thấy bạn này chỉ toàn hỏi bài nhỉ?

Mình thắc mắc nên mới hỏi thôi bạn, với mình cũng không chuyên lắm nên mình cũng đâu đi giải bài khác được bạn. Mình nghĩ diễn đàn lập ra để trao đổi, những người chưa biết thì phải hỏi thôi bạn?




#738047 $\frac{a^2+b^2+c^2}{ab+bc+ca}+\frac{8...

Posted by Math04 on 25-03-2023 - 17:43 in Bất đẳng thức - Cực trị

Việc hỏi bài trên diễn đàn không có gì sai nhé các bạn, cứ mạnh dạn đăng bài nếu cần sự giúp đỡ. Nhưng cũng có thể @chuyenndu không phải chỉ trích gì mà chỉ có ý khuyến khích @Math04 tham gia thảo luận hơn mà thôi, em nên xem đó như là một lời góp ý.

 

Việc học trên diễn đàn nếu chỉ hỏi bài thôi thì hơi uổng. @Math04 thử tìm những bài chưa giải (có rất nhiều) và thử giải xem sao, anh nghĩ là có nhiều bài vừa sức với em đấy, như thế sẽ học được nhiều hơn. Ngoài ra lúc em hỏi bài, thì cũng có thể trình bày xem là mình đã cố gắng làm như thế nào, bí chỗ nào, như thế thì mọi người có thể gợi ý cho em tự làm tiếp, sẽ hiệu quả hơn cho em đấy. Có thể thử bắt đầu với bài ở trên chẳng hạn  :like

Em cám ơn anh ạ




#736596 Chứng minh tồn tại số nguyên dương $M$ sao cho: $a_{m...

Posted by Math04 on 01-01-2023 - 17:25 in Số học

Bạn có kinh nghiệm gì khi mà tìm bài trên AOPS không nhỉ kiểu gõ công thức hay là gõ đề vào tại nhiều lúc mình tìm mà không ra. Với tìm ngay chỗ Search Community hay là Advanced Community Search nhỉ




#736579 Chứng minh tồn tại số nguyên dương $M$ sao cho: $a_{m...

Posted by Math04 on 31-12-2022 - 21:19 in Số học

Đặt $s_n = \frac{a_1}{a_2} + \frac{a_2}{a_3}+...+\frac{a_{n-1}}{a_n}+\frac{a_n}{a_1}$.

Ta có $s_n\in\mathbb Z,\forall n\geq N$ nên $s_{n+1} - s_n\in\mathbb Z,\forall n\geq N$

$\Rightarrow \frac{a_n}{a_{n+1}} + \frac{a_{n+1}}{a_1} - \frac{a_n}{a_1}\in\mathbb Z,\forall n\geq N$.

Lấy $p$ là số nguyên tố bất kì. 

Nhận xét: Với mọi $n\geq N$: Nếu $v_p(a_n)> v_p(a_1)$ thì $v_p(a_{n+1})> v_p(a_1)$. Ngược lại, nếu $v_p(a_n)\leq v_p(a_1)$ thì $v_p(a_{n+1})\leq v_p(a_1)$

Chứng minh: Giả sử phản chứng.

TH1: $v_p(a_n)> v_p(a_1)$: Khi đó theo giả sử thì $v_p(a_{n+1}) \leq v_p(a_1) < v_p(a_n)$

$\Rightarrow v_p\left(\frac{a_{n+1}}{a_1}\right) < v_p\left(\frac{a_n}{a_1}\right) \leq v_p\left(\frac{a_n}{a_{n+1}}\right)$

$\Rightarrow v_p\left(\frac{a_n}{a_{n+1}} + \frac{a_{n+1}}{a_1} - \frac{a_n}{a_1}\right) = v_p\left(\frac{a_{n+1}}{a_1}\right) < 0$, vô lí vì $\frac{a_n}{a_{n+1}} + \frac{a_{n+1}}{a_1} - \frac{a_n}{a_1}\in\mathbb Z$.

TH2: $v_p(a_n) \leq v_p(a_1)$: Khi đó theo giả sử thì $v_p(a_n) \leq v_p(a_1) < v_p(a_{n+1})$

$\Rightarrow v_p\left(\frac{a_n}{a_{n+1}}\right) < v_p\left(\frac{a_n}{a_1}\right) <v_p\left(\frac{a_{n+1}}{a_1}\right)$

$\Rightarrow v_p\left(\frac{a_n}{a_{n+1}} + \frac{a_{n+1}}{a_1} - \frac{a_n}{a_1}\right) = v_p\left(\frac{a_n}{a_{n+1}}\right) < 0$, vô lí vì $\frac{a_n}{a_{n+1}} + \frac{a_{n+1}}{a_1} - \frac{a_n}{a_1}\in\mathbb Z$.

Từ nhận xét trên, ta thấy $v_p(a_n) > v_p(a_1),\forall n\geq N$ hoặc $v_p(a_n) \leq v_p(a_1),\forall n \geq N$.

$\bullet$ Nếu $v_p(a_n) > v_p(a_1),\forall n\geq N$ thì $v_p(a_n)\geq v_p(a_{n+1}),\forall n\geq N$.

Thật vậy, do $v_p\left(\frac{a_{n+1}}{a_1} - \frac{a_n}{a_1}\right) >0,\forall n\geq N$ và $v_p\left(\frac{a_n}{a_{n+1}} + \frac{a_{n+1}}{a_1} - \frac{a_n}{a_1}\right),\forall n\geq N$ nên $v_p\left(\frac{a_n}{a_{n+1}}\right)\geq 0,\forall n\geq N$

$\Rightarrow v_p(a_n)\geq v_p(a_{n+1}),\forall n\geq N$.

$\bullet$ Nếu $v_p(a_n)\leq v_p(a_1),\forall n\geq N$ thì ta vẫn có $v_p(a_n)\geq v_p(a_{n+1}),\forall n\geq N$.

Thật vậy, giả sử tồn tại $n\geq N$ sao cho $v_p\left(a_{n+1}\right) > v_p\left(a_{n}\right)$.

Thế thì $v_p\left(\frac{a_{n+1}}{a_1}\right) < v_p\left(\frac{a_n}{a_1}\right) \leq v_p\left(\frac{a_n}{a_{n+1}}\right)\Rightarrow v_p\left(\frac{a_n}{a_{n+1}} + \frac{a_{n+1}}{a_1} - \frac{a_n}{a_1}\right) = v_p\left(\frac{a_{n+1}}{a_1}\right) < 0$, vô lí.

Do đó dãy $u_n = v_p(a_n)$ là dãy không tăng kể từ $N$. Mặt khác, dãy này là dãy số nguyên không âm nên đến một lúc nào đó, dãy là hằng.

Điều này đúng với mọi số nguyên tố $p$ nên suy ra $(a_n)$ là hằng số kể từ chỉ số nào đó. (đpcm)

Cám ơn bạn nhé làm sao để nghĩ ra lời giải như vầy nhỉ




#734232 Chứng minh $d(n) \leq 2\sqrt{n}$ với mọi số ngu...

Posted by Math04 on 05-08-2022 - 23:39 in Số học

Mình nhầm đó bạn, với mọi $p_i\geq 5$ thì đánh giá chặt $p_1^{k_1} > (p_1+1)^2$ luôn.
Với $p_i\in\{2;3\}$ mà $k_i \geq 3$ thì cũng đánh giá chặt được như vậy.
Việc còn lại là thử.

Bạn có thể trình bày chi tiết giúp mình với



#736504 Chứng minh tồn tại số nguyên dương $M$ sao cho: $a_{m...

Posted by Math04 on 30-12-2022 - 10:35 in Số học

Xét dãy $(a_{n})$ gồm vô hạn các số nguyên dương. Gỉa sử tồn tại số nguyên dương $N>1$ sao cho với mỗi $n\geq N$ thì $\frac{a_{1}}{a_{2}}+\frac{a_{2}}{a_{3}}+...+\frac{a_{n-1}}{a_{n}}+\frac{a_{n}}{a_{1}}$ là số nguyên. Chứng minh tồn tại số nguyên dương $M$ sao cho: $a_{m} = a_{m+1}, \forall m \geq M$.




#734190 Chứng minh $d(n) \leq 2\sqrt{n}$ với mọi số ngu...

Posted by Math04 on 03-08-2022 - 22:10 in Số học

Với mỗi số nguyên dương $n$, kí hiệu $d(n)$ là số các ước nguyên dương của $n$. Chứng minh $d(n) \leq 2\sqrt{n}$ với mọi số nguyên dương $n$.



#734213 Chứng minh $d(n) \leq 2\sqrt{n}$ với mọi số ngu...

Posted by Math04 on 04-08-2022 - 22:15 in Số học

Viết $n = p_1^{k_1} . p_2^{k_2} ... p_h^{k_h}$.

Khi đó $d(n) = (k_1+1)(k_2+1)...(k_h+1)$.

Ta sẽ chứng minh: $k_1+1 \leq 2\sqrt{p_1^{k_1}}$ hay $(k_1+1)^2 \leq 4p_1^{k_1}$.

Nhận thấy VT là một đa thức bậc hai, còn VP là một luỹ thừa nên bất đẳng thức này dễ dàng chứng minh bằng quy nạp.

Tương tự với $k_2,...,k_h$ ta có đpcm.

Bạn cho mình hỏi nếu làm như vậy thì khi ta nhân các bất đẳng thức đó lại thì sẽ được $d(n) \leq 2^h\sqrt{n}$ thì vẫn chưa có điều cần chứng minh mà bạn. 




#735790 Tồn tại đa thức bậc $ n$ có hệ số nguyên $p ( x )$ sao ch...

Posted by Math04 on 20-11-2022 - 15:46 in Đa thức

Chứng minh rằng với mỗi số nguyên dương $n$ ,tồn tại đa thức bậc $ n$ có hệ số nguyên $p ( x )$ sao cho $p ( 0 ) , p ( 1 ) , . . . , p ( n )$  là các số nguyên dương đôi một khác nhau,và tất cả chúng đều có dạng $2a^k + 3$




#735391 Tìm số các từ có độ dài $n$ trên một từ điển có 3 chữ cái $...

Posted by Math04 on 20-10-2022 - 21:23 in Tổ hợp và rời rạc

Với $n$ là số nguyên dương cho trước, tìm số các từ có độ dài $n$ trên một từ điển có 3 chữ cái ${a,b,c}$ sao cho chữ cái $a$ xuất hiện một số lẻ lần.




#735393 Tìm số các dãy $a_{1},a_{2},a_{3},...,a_{n}$ thỏa điều kiện đã cho...

Posted by Math04 on 20-10-2022 - 21:32 in Tổ hợp và rời rạc

Cho dãy $a_{1},a_{2},a_{3},...,a_{n}$ sao cho $a_{i} \in \left \{ 0,1,2 \right \}$ và $\left | a_{i}-a_{i+1} \right |\leq 1, \forall i$. Tìm số các dãy $a_{1},a_{2},a_{3},...,a_{n}$ thỏa điều kiện đã cho.




#736575 Chứng minh rằng tồn tại số nguyên $a$, $1<a<\frac...

Posted by Math04 on 31-12-2022 - 14:02 in Số học

Bài này nhìn tưởng đơn giản nhưng để thỏa mãn điều kiện bất đẳng thức thì rắc rối ghê  :wacko:

 

Đặt phân tích ra thừa số nguyên tố của $n$ là $p_1^{\alpha_1}p_2^{\alpha_2}\dots p_k^{\alpha_k}$. Theo định lí thặng dư Trung Hoa, với mỗi $i\in \{1,2,\dots,k\}$ thì tồn tại số nguyên $x_i$ thỏa mãn

\[\left\{\begin{array}{l}x_i\equiv 1\pmod{p_i^{\alpha_i}}\\ x_i\equiv 0\pmod{\frac{n}{p_i^{\alpha_i}}}\end{array}\right..\]

Nhận xét (NX). Cho tổng $X=\sum_{i=1}^k\epsilon_ix_i$, trong đó $\epsilon_i\in \{0,1\}$ với mọi $i$.

  1. $n\mid X^2-X$.
  2. Nếu $X$ chia $n$ dư $0$ hoặc $1$ thì $\epsilon_1=\epsilon_2=\dots=\epsilon_k$.

 

Với mỗi $i\in \{1,2,\dots,k\}$, gọi $X_i$ là số tự nhiên nhỏ nhất thỏa mãn

\[X_i\equiv x_1+x_2+\dots+x_i\pmod{n}.\]

Theo NX2 thì $X_i\neq 0$ với mọi $i=\overline{1,k}$ và ngoài ra $X_k=1$. Bổ sung thêm $X_0=0$. Xét $k+1$ số $X_0,\ X_1,\ \dots,\ X_k$, theo định lí Dirichlet sẽ tồn tại $0\le c<d\le k$ sao cho

\[X_c,\ X_d\in\left [ \frac{jn}{k},\frac{(j+1)n}{k} \right ).\]

Nghĩa là $|X_d-X_c|<\frac{n}{k}$, theo NX2 ta có $X_d-X_c\notin \{0,1\}$. Cuối cùng ta chia ra hai trường hợp sau

$\bullet$ TH1: $X_d-X_c>1$ thì đặt

$$a_1=x_{c+1}+x_{c+2}+\dots+x_d=X_d-X_c\in\left(1,\frac{n}{k}\right).$$

Ngoài ra theo NX1 thì $n\mid a_1^2-a_1$ nên $a_1$ thỏa đề.

$\bullet$ TH2: $X_d-X_c<0$ thì đặt

$$a_2=(x_1+x_2+\dots+x_c)+(x_{d+1}+x_{d+2}+\dots+x_k)=X_k-(X_d-X_c)\in\left(1,1+\frac{n}{k}\right).$$

Ngoài ra theo NX1 thì $n\mid a_2^2-a_2$ nên $a_2$ thỏa đề.

À bạn giải thích rõ chỗ "theo định lí Dirichlet sẽ tồn tại $0\le c<d\le k$ sao cho

\[X_c,\ X_d\in\left [ \frac{jn}{k},\frac{(j+1)n}{k} \right ).\]" giúp mình với. Bạn áp dụng nguyên lí định lí Dirichlet như thế nào nhỉ




#735483 Chứng minh rằng tồn tại số nguyên $a$, $1<a<\frac...

Posted by Math04 on 26-10-2022 - 22:02 in Số học

Cho $n$ là số nguyên lớn hơn $1$, $k$ là số các ước nguyên tố phân biệt của $n$. Chứng minh rằng tồn tại số nguyên $a$, $1<a<\frac{n}{k}+1$ sao cho $n|(a^2-a)$.



#736246 Chứng minh rằng tồn tại số nguyên $a$, $1<a<\frac...

Posted by Math04 on 15-12-2022 - 11:18 in Số học

Bài này nhìn tưởng đơn giản nhưng để thỏa mãn điều kiện bất đẳng thức thì rắc rối ghê  :wacko:

 

Đặt phân tích ra thừa số nguyên tố của $n$ là $p_1^{\alpha_1}p_2^{\alpha_2}\dots p_k^{\alpha_k}$. Theo định lí thặng dư Trung Hoa, với mỗi $i\in \{1,2,\dots,k\}$ thì tồn tại số nguyên $x_i$ thỏa mãn

\[\left\{\begin{array}{l}x_i\equiv 1\pmod{p_i^{\alpha_i}}\\ x_i\equiv 0\pmod{\frac{n}{p_i^{\alpha_i}}}\end{array}\right..\]

Nhận xét (NX). Cho tổng $X=\sum_{i=1}^k\epsilon_ix_i$, trong đó $\epsilon_i\in \{0,1\}$ với mọi $i$.

  1. $n\mid X^2-X$.
  2. Nếu $X$ chia $n$ dư $0$ hoặc $1$ thì $\epsilon_1=\epsilon_2=\dots=\epsilon_k$.

 

Với mỗi $i\in \{1,2,\dots,k\}$, gọi $X_i$ là số tự nhiên nhỏ nhất thỏa mãn

\[X_i\equiv x_1+x_2+\dots+x_i\pmod{n}.\]

Theo NX2 thì $X_i\neq 0$ với mọi $i=\overline{1,k}$ và ngoài ra $X_k=1$. Bổ sung thêm $X_0=0$. Xét $k+1$ số $X_0,\ X_1,\ \dots,\ X_k$, theo định lí Dirichlet sẽ tồn tại $0\le c<d\le k$ sao cho

\[X_c,\ X_d\in\left [ \frac{jn}{k},\frac{(j+1)n}{k} \right ).\]

Nghĩa là $|X_d-X_c|<\frac{n}{k}$, theo NX2 ta có $X_d-X_c\notin \{0,1\}$. Cuối cùng ta chia ra hai trường hợp sau

$\bullet$ TH1: $X_d-X_c>1$ thì đặt

$$a_1=x_{c+1}+x_{c+2}+\dots+x_d=X_d-X_c\in\left(1,\frac{n}{k}\right).$$

Ngoài ra theo NX1 thì $n\mid a_1^2-a_1$ nên $a_1$ thỏa đề.

$\bullet$ TH2: $X_d-X_c<0$ thì đặt

$$a_2=(x_1+x_2+\dots+x_c)+(x_{d+1}+x_{d+2}+\dots+x_k)=X_k-(X_d-X_c)\in\left(1,1+\frac{n}{k}\right).$$

Ngoài ra theo NX1 thì $n\mid a_2^2-a_2$ nên $a_2$ thỏa đề.

Bạn có thể chia sẻ quá trình nghĩ ra lời giải này không nhỉ? Cám ơn bạn nhé!




#732758 $\frac{(x-4)\sqrt{x-2}-1}{\sqrt...

Posted by Math04 on 27-02-2022 - 13:18 in Phương trình - Hệ phương trình - Bất phương trình

Đánh giá hai vế tìm được $x=3$.

Cụ thể là đánh giá như thế nào vậy bạn




#734994 $\frac{P(x^2+1)}{x^2+1}=\frac{P(x^2+2...

Posted by Math04 on 18-09-2022 - 00:51 in Đa thức

Tìm tất cả các đa thức $P(x)$ hệ số thực thỏa:

$\frac{P(x^2+1)}{x^2+1}=\frac{P(x^2+2)}{x^2+2}, \forall x \in \mathbb{R}$




#734539 Tìm số $k$ nguyên dương nhỏ nhất để tạo thành tập hợp đẹp gồm...

Posted by Math04 on 20-08-2022 - 18:45 in Tổ hợp và rời rạc

Một tập hợp các đường thẳng trong mặt phẳng được gọi là đẹp nếu các đường thẳng trong tập hợp này phân biệt, không có $3$ đường nào đồng quy và mỗi đường thẳng trong chúng đều có số các giao điểm của nó với tất cả các đường thẳng còn lại là một số lẻ. Tìm số $k$ nguyên dương nhỏ nhất sao cho với $2018$ đường thẳng phân biệt, không có $3$ đường nào đồng quy, bất kỳ, ta có thể bổ sung thêm $k$ đường thẳng để tạo thành tập hợp đẹp gồm $2018+ k$ đường thẳng.




#735041 $\frac{P(x^2+1)}{x^2+1}=\frac{P(x^2+2...

Posted by Math04 on 20-09-2022 - 22:59 in Đa thức

Nhân chéo ta có $(x^2+2)P(x^2+1)=(x^2+1)P(x^2+2)$ với mọi $x$ thuộc $\mathbb{R}$. Xem $P(x)$ như một đa thức hệ số trên $\mathbb{C}$ ta thấy đẳng thức trên cũng đúng với mọi $x$ thuộc $\mathbb{C}$. Cho $x^2+1=0$ ta thấy $P(0)=0$ hay $x \mid P(x)$. Nói cách khác tồn tại $Q(x) \in \mathbb{R}[x]$ mà $P(x)=xQ(x)$. Từ đây ta thấy $Q(x^2+1)=Q(x^2+2)$ với mọi $x$ thuộc $\mathbb{C}$. Nếu $Q = 0$ thì $P=0$. Nếu $Q \neq 0$ thì $Q$ có ít nhất một nghiệm phức $a$, khi đó dễ thấy (do phương trình $x^2+1=a$ luôn có nghiệm) $a+1$ cũng là nghiệm. Lặp lại quá trình này ta thấy $Q$ có vô số nghiệm, vô lý. Vậy chỉ có $P=0$ thoả mãn.

Mình thắc mắc chỗ "Xem $P(x)$ như một đa thức hệ số trên $\mathbb{C}$ ta thấy đẳng thức trên cũng đúng với mọi $x$ thuộc $\mathbb{C}$, bạn có thể giải thích cụ thể sao đẳng thức trên cũng đúng với mọi $x$ thuộc $\mathbb{C}$ giúp mình với




#732659 $\frac{(x-4)\sqrt{x-2}-1}{\sqrt...

Posted by Math04 on 12-02-2022 - 22:07 in Phương trình - Hệ phương trình - Bất phương trình

Giải phương trình: $\frac{(x-4)\sqrt{x-2}-1}{\sqrt{4-x}+x-5}=\frac{2+(2x-4)\sqrt{x-2}}{x-1}$




#734161 Tìm giới hạn của $u_{n}$

Posted by Math04 on 01-08-2022 - 20:55 in Dãy số - Giới hạn

Bổ đề. Cho hai số dương $\alpha,\beta$ có tổng bé hơn $1$. Nếu dãy số $(x_n)$ dương thỏa mãn 

$$x_{n+2}\le \alpha x_{n+1}+\beta x_{n},\quad \forall n\ge n_0$$

thì dãy $(x_n)$ hội tụ và $\lim x_n=0$.

 

Quay lại bài toán.

Dễ dàng chứng minh được $0<u_n<1$ với mọi $n$. Sử dụng bất đẳng thức Cô-si ta có 

$$u_{n+2}=\frac{1}{n}u_{n+1}^2+\frac{n-1}{n}\sqrt{u_n}\ge \sqrt[n]{u_{n+1}^2u_n^{\frac{n-1}{2}}}.$$

Đặt $v_n=-\ln(u_n)>0$ thì bất đẳng thức trên tương đương $v_{n+2}\le \frac{2}{n}v_{n+1}+\frac{n-1}{2n}v_n$. Suy ra

$$v_{n+2}<\frac{2}{5}v_{n+1}+\frac{1}{2}v_n,\quad \forall n\ge 5.$$

Áp dụng bổ đề trên ta có $\lim v_n=0$, dẫn tới $\lim u_n=1$.

 

P/s: Một ví dụ và tài liệu tham khảo liên quan tới bổ đề trên có thể xem ở đây

Bạn có thể nói rõ chỗ lấy $ln$ làm sao ra được như vậy được không bạn thì mình chưa học tới $ln$ á bạn. Mình chỉ biết sơ về định nghĩa chứ chưa hiểu sâu cụ thể làm sao lấy $ln$ thì ra được vậy mong bạn giúp mình với




#738039 Tính $a_n$ là số cách điền thỏa ba ô liên tiếp nhau bất kỳ đều khôn...

Posted by Math04 on 25-03-2023 - 15:19 in Tổ hợp và rời rạc

Ta gọi một cách điền "rất đẹp" nếu như đó là cách điền đẹp và có hai ô cuối cùng có số giống nhau.

Thế thì gọi $b_n$ là số cách điền "rất đẹp".

+) Tính $a_{n+1}$: Với mỗi cách điền $n$ ô trước là "đẹp" nhưng không phải "rất đẹp" ta có $2$ cách điền ô thứ $n+1$. Đồng thời với mỗi cách điền $n$ ô trước "không đẹp" ta có duy nhát $1$ cách điền ô thứ $n=1$. Do đó $a_{n+1} = 2a_n - b_n$.

+) Tính $b_{n+1}$: Giả sử ô thứ $n$ và $n+1$ đều là $0$. Thế thì ô thứ $n-1$ là $1$, và số cách điền lúc này chính là số cách điền $n-2$ ô đầu tiên có hai vị trí cuối không cùng bằng $1$, và bằng $a_{n-2} - \frac{b_{n-2}}{2}$.

Suy ra $b_{n+1} = 2a_{n-2} - b_{n-2}$.

Kết hợp lại ta có công thức truy hồi: $\begin{cases} a_{n+1} = 2a_n - b_n \\ b_{n+1} = 2a_{n-2} - b_{n-2}\end{cases}$

$\Rightarrow b_n = 2a_n - a_{n+1},\forall n\geq 1\Rightarrow 2a_{n+1} - a_{n+2} = 2a_{n-2} - 2a_{n-2} + a_{n-1},\forall n\geq 3$

$\Rightarrow a_{n+2} = 2a_{n+1} - a_{n-1},\forall n\geq 3$.

Tính được: $a_1 = 2, a_2 = 4, a_3 = 6, a_4 = 10$. Do đó ta có $a_{n+3} = 2a_{n+2} - a_n,\forall n\in\mathbb N^*$.

Từ đây dễ dàng tìm được CTTQ của $(a_n)$.

Hoàng có thể chia sẻ cách em học các phân môn không nhỉ cũng như từng tài liệu/ nguồn bài mà em tham khảo




#737953 Tính $a_n$ là số cách điền thỏa ba ô liên tiếp nhau bất kỳ đều khôn...

Posted by Math04 on 23-03-2023 - 10:43 in Tổ hợp và rời rạc

Xét hình chữ nhật $1 \times n$ gồm $n$ ô vuông $1 \times 1$. Mỗi ô điền số $0$ hoặc $1$. Một cách điền "đẹp" nếu như ba ô liên tiếp nhau bất kỳ đều không chứa ba số giống nhau. Với mỗi $n \geq 3$, gọi $a_n$ là số cách điền "đẹp". Tính $a_n$.




#734214 Tìm tất cả các số nguyên dương $n$ sao cho dãy số $(u_{k...

Posted by Math04 on 04-08-2022 - 22:18 in Số học

Với mỗi số nguyên dương $n$, kí hiệu $d(n)$ là số các ước nguyên dương của $n$ và $u_{1}=d(n), u_{k+1}=d(u_{k})$, $k=1,2,3,...$.
a) Chứng minh $d(n) \leq 2\sqrt{n}$ với mọi số nguyên dương $n$.
b) Tìm tất cả các số nguyên dương $n$ sao cho dãy số $(u_{k})$, $k=1,2,3,...$ không chứa bất kì số chính phương nào.